============================================================ クーロンポテンシャルのフーリエ変換 ============================================================ 結構、有名な積分だと思います。 ときどき解法を忘れてしまうので、自分用にメモです。 問題の積分は、 I &= \int \dfrac{e^{-i\bm{k} \ \cdot \ \bm{r}}}{|\bm{r}|} d \bm{r} \\ &= \int_{-\infty}^\infty \int_{-\infty}^\infty \int_{-\infty}^\infty \dfrac{exp(-i(k_x x + k_y y + k_z z))} {\sqrt{x^2+y^2+z^2}} \ dxdydz \tag{##} 式 $(1)$ の第二行を見るとなんとも物騒な積分ですが、この積分は次の極座標を用いれば簡単になります。 x = r \sin \theta \cos \phi \\ y = r \sin \theta \sin \phi \\ z = r \cos \theta ちなみに $|\bm{r}|=r$ です。一応初めての方もいらっしゃると思うので、 この変換のヤコビアン(ヤコビの行列式:積分の微小体積要素の変換式)は、お馴染み(?)の dx dy dz &= \begin{vmatrix} \dfrac{dx}{dr} & \dfrac{dx}{d\theta} & \dfrac{dx}{d \phi} \\ \dfrac{dy}{dr} & \dfrac{dy}{d\theta} & \dfrac{dy}{d \phi} \\ \dfrac{dz}{dr} & \dfrac{dz}{d\theta} & \dfrac{dz}{d \phi} \\ \end{vmatrix} dr d \theta d \phi\\ &= \begin{vmatrix} \sin \theta \cos \phi & r \cos \theta \cos \phi & - r \sin \theta \sin \phi \\ \sin \theta \sin \phi & r \cos \theta \sin \phi & r \sin \theta \cos \phi \\ \cos \theta & - r \sin \theta & 0 \end{vmatrix} dr d \theta d \phi \\ &= (r^2 \sin^3 \theta \cos^2 \phi + r^2 \cos^2 \theta \sin \theta \cos^2 \phi \\ &+ r^2 \sin^3 \theta \sin^2 \phi + r^2 \cos^2 \theta \sin \theta \sin^2 \phi ) dr d \theta d \phi \\ &= (r^2 \sin^3 \theta + r^2 \cos^2 \sin \theta ) dr d \theta d \phi \\ &= r^2 \sin \theta dr d \theta d \phi \tag{##} です。よって、 $\bm{k}$ と $\bm{r}$ が $\theta$ の角を成すとして、 うまく座標系の取り方を工夫して、 $\bm{r}$ を $z$ 方向を向いたベクトルとすれば、 I &= \int \dfrac{e^{-i\bm{k} \ \cdot \ \bm{r}}}{|\bm{r}|} d \bm{r} \\ &= \int_{0}^\infty dr \int_0^{\pi} d \theta \int_0^{2\pi}d \phi \dfrac{e^{- i k r \cos \theta}}{ r } r^2 \sin \theta \tag{##} ここで、 $\cos \theta = t$ , $ - \sin \theta d \theta = dt $ と変数変換すると、 I &= \int \dfrac{e^{-i\bm{k} \ \cdot \ \bm{r}}}{|\bm{r}|} d \bm{r} \\ &= \int_{0}^\infty dr \int_0^{2\pi}d \phi \left( \int_{-1}^{1} r e^{- i k r t} dt \right) \\ &= \int_{0}^\infty dr \int_0^{2\pi}d \phi \left[ r \frac{e^{- i k r t} }{-ikr} \right]_{-1}^{1} \\ &= \int_{0}^\infty dr \int_0^{2\pi}d \phi \left[ \frac{e^{- i k r t} }{ik} \right]_{1}^{-1} \\ &= \int_{0}^\infty dr \int_0^{2\pi}d \phi \dfrac{1}{ik} \left( e^{i k r } - e^{- i k r } \right) \\ &= \int_{0}^\infty dr \int_0^{2\pi}d \phi \dfrac{2}{k} \sin (kr) \tag{##} ここでいかにも物理(not数学的な意味で)らしい手法を 用います。 $\lim_{\delta \to 0 }e^{- \delta r} (= 1) $ を被積分関数に掛けるのです。 すると、無限遠での値が収束し、 I &= \lim_{\delta \to 0} \int_{0}^\infty dr \int_0^{2\pi}d \phi \dfrac{1}{ik} e^{-\delta r} (e^{i kr} - e^{-ikr}) \\ &= \int_0^{2 \pi} d \phi \dfrac{1}{ik} \lim_{\delta \to 0} \left[ \dfrac{e^{(ik-\delta)r}}{ik-\delta} - \dfrac{e^{(-ik-\delta)r}}{-ik-\delta} \right]_{0}^{\infty} \\ &= \int_0^{2 \pi} d \phi \dfrac{1}{ik} \lim_{\delta \to 0} \left[ \dfrac{ 0-1 }{ik-\delta} + \dfrac{ 0-1 }{ik+\delta} \right] \\ &= \int_0^{2 \pi} d \phi \dfrac{1}{ik} \left( \dfrac{-2}{ik} \right) \\ &= \int_0^{2 \pi} d \phi \dfrac{2}{k^2} \\ &= \dfrac{4 \pi}{k^2} \tag{##} となります。よって、 \int \dfrac{e^{-i\bm{k} \ \cdot \ \bm{r}}}{|\bm{r}|} d \bm{r} &= \dfrac{4 \pi}{k^2} \tag{##} が言えました。 それでは今日はこの辺で、お疲れ様でした。 @@author:クロメル@@ @@accept:2011-06-14@@ @@category:フーリエ解析@@ @@id:coulombFourier@@